LSAT 12 – Section 4 – Question 13

You need a full course to see this video. Enroll now and get started in less than a minute.

Target time: 2:26

This is question data from the 7Sage LSAT Scorer. You can score your LSATs, track your results, and analyze your performance with pretty charts and vital statistics - all with a Free Account ← sign up in less than 10 seconds

Question
QuickView
Type Tags Answer
Choices
Curve Question
Difficulty
Psg/Game/S
Difficulty
Explanation
PT12 S4 Q13
+LR
PSA - Find the rule +PSAr
A
9%
162
B
10%
160
C
52%
167
D
19%
161
E
10%
163
152
164
175
+Hardest 149.171 +SubsectionMedium

This is a pseudo-sufficient assumption question because of the stem: Which one of the following principles... supports the view that a physician’s responding to the request would violate medical ethics? This is at once a PSAr question and a

The stimulus tells us that the police have a wanted poster in a medical journal for a specific fugitive with a non-infectious skin condition that would eventually require him to visit a doctor, asking for information on where the fugitive is. Because doctors are required to report gunshot wounds to police and certain infectious disease to the health authorities, reporting information about this fugitive, according to the argument, would not violate medical ethics. Our author concludes that these exceptions to confidentiality, meaning sharing information on our fugitive, are clearly ethical.

if we read those requirements again very closely, we’ll see that the conclusion does not follow logically from this. Based on the requirement (report to police if it’s a gunshot wound OR health authorities if it’s certain types of infectious diseases), the physician in our stimulus shouldn’t report our fugitive since he has neither a gunshot wound nor an infectious disease.

Figuring out what the question stem is asking us to do is extremely important here. Since our question stem is essentially asking us to support a view that disagrees with the stimulus, our correct answer choice will weaken the argument that the doctor is not violating medical ethics. In other words, we’re trying to weaken the idea that a doctor reporting to the authorities with information on the fugitive is an ethical exception to confidentiality.

Answer Choice (A) This rule brings forward requirements as citizens to report certain issues to the authorities, and nothing about the doctor violating medical ethics. This doesn’t weaken our conclusion by way of a rule/assumption.

Answer Choice (B) We still have to observe the requirements put forward in the stimulus; this answer choice would go against our requirements.

Correct Answer Choice (C) This is correct because it’s pointing out that unless the fugitive had a gunshot wound (thereby requiring the doctor to report the fugitive to the police), the doctor should not be sharing information on the fugitive with anyone (which would then reduce their willingness to come in to be treated for fear of being detained).

Answer Choice (D) Similar to answer choice (B), this would go against the requirement laid out in the stimulus.

Answer Choice (E) Again, we are given exceptions to confidentiality (gunshot wounds and infectious diseases), so this answer choice goes against this requirement, much like answer choices B and D do.

Take PrepTest

Review Results

Leave a Reply